LSAT and Law School Admissions Forum

Get expert LSAT preparation and law school admissions advice from PowerScore Test Preparation.

 Administrator
PowerScore Staff
  • PowerScore Staff
  • Posts: 8916
  • Joined: Feb 02, 2011
|
#34826
Complete Question Explanation

Flaw in the Reasoning—CE. The correct answer choice is (A)

This political scientist says that if people believe that problems can only be solved by big groups of people changing their minds about something (and that type of change doesn’t normally arise from government action), those people become unenthusiastic about voting:
  • Cause ..... ..... ..... ..... ..... ..... ..... ..... ..... ..... Effect
    Believe important problems solved by large ..... :arrow: ..... unenthusiastic about voting
    attitude shifts (not government action)
Based on this premise the author concludes that decreasing voter turnout must be completely caused by a strengthening belief that important problems are not changed by government action.

The problem reflected in this argument is that the author jumps from a premise about a possible cause (the belief that important problems are solved by large attitude shifts, not government action), to the conclusion that this must be the cause.

Not surprisingly, this choice is followed by a Flaw question, so the correct answer choice will present the flaw discussed above. Note also that the correct answer choice when dealing with a Cause-Effect flaw is likely to include terms such as "cause," "effect," or synonymous terms.

Answer choice (A): This is the correct answer choice. It provides that the author lacks justification for the presumption that the referenced belief is the only cause there is for decreasing voter turnout (note also that this is the only choice that features the term "cause").

Answer choice (B): The stimulus deals with the belief that important problems cannot be addressed with government action, and how that can decrease enthusiasm for voting. This is not the author's presumption, and it is not the argument's flaw.

Answer choice (C): This choice presents a flaw that is different from the answer prephrased above, so this cannot be the correct answer to this Flaw question.

Answer choice (D): The author does not undermine the claim that people don't believe that important problems can be addressed via political means. On the contrary, the author presumes this to be the cause of decreasing voter turnout.

Answer choice (E): The author does not presume that voter apathy prevents attitudinal changes, the author attributes voter apathy to an increasing belief in the need for attitudinal changes to address important problems.
 mkuo
  • Posts: 24
  • Joined: Nov 06, 2012
|
#11507
Hi,

As wordy and incoherent as the stimulus is, I think I get the gist of it:
People believe that important problems can be addressed only by large number of people drastically changing their attitudes AND such attitudinal changes generally do not result from government action ---> People are unenthusiastic about voting.

In short, people believe government action cannot address important questions ---> People are unenthusiastic about voting.

A conclusion was drawn from that:
This growing conviction of politicians cannot solve the most important problems causes decreasing voter turnout.

Now i understand clearly what the problems are with this stimulus: that the conclusion was TOO EXTREME and leaves out other possible causes for the turnout. But I can't make the connection between the belief in (A) and the one in the stimulus.

(A) states decreasing voter turnout is caused by the "belief that few important problems can be solved by government action."

Stimulus states the belief as the growing conviction of politicians cannot solve the most important problems.

How do you equate "cannot solve the most important problems" with "few important problems can be solved"?

Thanks in advance!
 Steve Stein
PowerScore Staff
  • PowerScore Staff
  • Posts: 1153
  • Joined: Apr 11, 2011
|
#11539
Hi,

That's a challenging question, and you have broken it down well. To review:

This political scientist says that if people believe that problems can only be solved by big groups of people changing their minds about something (and that type of changed doesn’t normally arise from government action), those people become unenthusiastic about voting:

Cause :arrow:
Believe important problems changed by large attitude shifts (not government action)

:arrow: Effect
unenthusiastic about voting

Based on this premise the author concludes that decreasing voter turnout must be completely caused by a strengthening belief that important problems are not changed by government action. The problem reflected in this argument is that the author jumps from a premise about a possible cause (the belief that important problems are changed by large attitude shifts, not government action), to the conclusion that this must be the cause.

Correct answer (A) provides that the author lacks justification for the presumption that the only possible cause, as you noted, is the referenced belief--but this refers to the belief that is presented in the first sentence of the stimulus, that "important problems can be addressed only by large numbers of people drastically changing their attitudes and that such attitudinal changes generally do not result from government action."

I hope that's helpful! Please let me know whether this is clear--thanks!

~Steve
 mkuo
  • Posts: 24
  • Joined: Nov 06, 2012
|
#11544
Hi Steve,

Thank you for your clarification. I was hoping if I could trouble you a bit more regarding your explanation.

Since "decreasing voter turnout" is caused by "the belief that politicians cannot solve the most important problems", and "unenthusiastic voters" is caused by "the belief that government generally cannot solve important problems", if you suggest that (A) is referring to the causal premise (govt generally cannot solve...), how did you make the connection between having "unenthusiastic voters" and "decreasing voter count"?

(A) is talking about "decreasing voters", and so is the conclusion, therefore when I thought in my previous post that (A) is referring to the conclusion, I didn't have that problem of involving the premise...

Without making any assumptions, the premise in my mind does not justify the conclusion. By referring (A) to the premise, how can I make that connection between the premise and the conclusion?
 Steve Stein
PowerScore Staff
  • PowerScore Staff
  • Posts: 1153
  • Joined: Apr 11, 2011
|
#11550
Thanks for your response.

Take another look at the conclusion: "The decreasing voter turnout is thus entirely due to..."

This stimulus is followed by a flaw question, so this one should be prephrased for sure. So, rather than seeking a link to something in one of the answer choices, how would you prephrase the flaw in the author's argument? (the italics are a hint)

Let me know--thanks!

~Steve
 mkuo
  • Posts: 24
  • Joined: Nov 06, 2012
|
#11555
Hi Steve,

I'd prephrase the flaw as "presumes, without justification, that there's no cause of decreasing voter turnout other than the belief that politicians cannot solve the most important problems."

I understand what the flaw is. In fact I think there are quite a few. As I mentioned in my first post, the obvious flaw has to do with calling one thing causes another as the only thing that could cause it without providing any support.

What I don't understand is where did political scientist say what (A) said: "the belief that few important problems can be solved by government action is the only cause of decreasing voter turnout"? People who get this question right infer that phrase from somewhere in the stimulus. But I fail to see it.

You brought it to my attention that "few important problems solved" is referred to the premise "... generally do not result from government action", but that is the cause of "unenthusiastic voters", not "decreasing voter turnout". Am I on the right track?
 Steve Stein
PowerScore Staff
  • PowerScore Staff
  • Posts: 1153
  • Joined: Apr 11, 2011
|
#11561
Thanks for your response. Again, the flaw, in broad terms, if I were to prephrase it, is that the author presents one possible cause and jumps to the conclusion that there is no other.

Going back to the stimulus, for further specificity, the author makes the following argument:

If there is a belief that important problems can be addressed only by large scale attitudinal change, and that such change does not generally result from government action, then people become unenthusiastic about voting.

Therefore, the entire cause of decreasing voter turnout (that is, the cause of decreased enthusiasm about voting) is the growing belief that politicians cannot solve the most important problems (that is, that such solutions do not generally come from government action--the action of politicians).

Cause ..... ..... ..... ..... ..... ..... ..... :arrow: Effect
Growing conviction in the above belief :arrow: decreasing enthusiasm/voter turnout

Then, as with many causal flaw questions, the answer to this one is that the author takes one possible cause and jumps to the conclusion that it is the only cause.

I hope this helps to clarify--please let me know--thanks!

~Steve
 mN2mmvf
  • Posts: 113
  • Joined: Jul 06, 2017
|
#37249
Hi,

Could you explain a bit more how the correct answer is (A)?

I interpreted the passage to mean:

if
People believe that important problems can be addressed only by large numbers of people changing their minds
(AND)
People believe that large numbers of people generally don't change their mind as a result of government action,
then=
People will be unenthusiastic about voting.

Which is to say, the government can't really do anything about solving important problems, because it can't change a large number of people's minds, and important problems can only be addressed by changing a large number of people's minds. That makes people unenthusiastic about voting, because what's the point?

The political scientist's conclusion about politicians seems to come from nowhere -- there is nothing in the conditional statement about politicians. To me, the flaw sounds like it is the fact that the political scientist is introducing an entirely new different potential cause that isn't supported in the argument.

Answer choice (A) says the flaw is that the political scientist presumes that there is no potential cause *except* the belief that few important problems can be solved by government action. But to me, it sounds like the political scientist is not committing that error at all. She is, after all, citing an entirely new and different possible cause -- politicians. That seems like the exact opposite of what (A) says. But is sounds exactly like choice (D), which is what I chose.
 Luke Haqq
PowerScore Staff
  • PowerScore Staff
  • Posts: 747
  • Joined: Apr 26, 2012
|
#37593
Hi mN2mmvf!

Happy to help explain (A) and (D) further.

I take your view to be that (D) is evidenced by the italicized part of the conclusion:
  • "The decreasing voter turnout is thus entirely due to a growing conviction that politicians cannot solve the most important problems."
By contrast, answer (A) claims the flaw is instead in the following italics:
  • "The decreasing voter turnout is thus entirely due to a growing conviction that politicians cannot solve the most important problems."
In my mind, the term "politicians" isn't really introducing new information but rather is referring to a subset of what it was already discussing. That is, the first sentence was about "government action," and what politicians do is a subset of that action. Even if it were a new element, answer (D) would seem to be a shell game. Answer (D) states that the flaw is that the argument,
  • "undermines its claim that people no longer believe there are political solutions to important problems by suggesting that people are dissatisfied with politicians."
Here, the italicized portion is what seems problematic to me. There's nothing in the stimulus about people no longer believing there are political solutions to important problems. People might very well believe there are political solutions to political problems--the stimulus instead focuses on their doubts about voting. If people (1) "believe that important problems can be addressed only by large numbers of people drastically changing their attitudes and (2) that such attitudinal changes generally do not result from government action," then people become unenthusiastic in voting, as you correctly diagram. But there's nothing stating those sufficient conditions (1) and (2) have occurred. In other words, we don't know whether or not people believe there are political solutions to important problems.

Answer (A) is right because of the italicized language "entirely due to" above. There's a big jump from the premises in the first sentence to the conclusion in the second, because even if the given reason for voter lack of enthusiasm is a concern, it doesn't justify the conclusion that this unenthusiastic attitude offers the complete explanation for decreasing voter turnout.
 dbpk
  • Posts: 16
  • Joined: May 07, 2017
|
#37755
Administrator wrote:Complete Question Explanation
  • Cause ..... ..... ..... ..... ..... ..... ..... ..... ..... ..... Effect
    Believe important problems solved by large ..... :arrow: ..... unenthusiastic about voting
    attitude shifts (not government action)
Hello! I was wondering why this line of reasoning was considered causal and not conditional

Get the most out of your LSAT Prep Plus subscription.

Analyze and track your performance with our Testing and Analytics Package.